matheraum.de
Raum für Mathematik
Offene Informations- und Nachhilfegemeinschaft

Für Schüler, Studenten, Lehrer, Mathematik-Interessierte.
Hallo Gast!einloggen | registrieren ]
Startseite · Forum · Wissen · Kurse · Mitglieder · Team · Impressum
Forenbaum
^ Forenbaum
Status Schulmathe
  Status Primarstufe
  Status Mathe Klassen 5-7
  Status Mathe Klassen 8-10
  Status Oberstufenmathe
    Status Schul-Analysis
    Status Lin. Algebra/Vektor
    Status Stochastik
    Status Abivorbereitung
  Status Mathe-Wettbewerbe
    Status Bundeswettb. Mathe
    Status Deutsche MO
    Status Internationale MO
    Status MO andere Länder
    Status Känguru
  Status Sonstiges

Gezeigt werden alle Foren bis zur Tiefe 2

Navigation
 Startseite...
 Neuerdings beta neu
 Forum...
 vorwissen...
 vorkurse...
 Werkzeuge...
 Nachhilfevermittlung beta...
 Online-Spiele beta
 Suchen
 Verein...
 Impressum
Das Projekt
Server und Internetanbindung werden durch Spenden finanziert.
Organisiert wird das Projekt von unserem Koordinatorenteam.
Hunderte Mitglieder helfen ehrenamtlich in unseren moderierten Foren.
Anbieter der Seite ist der gemeinnützige Verein "Vorhilfe.de e.V.".
Partnerseiten
Weitere Fächer:

Open Source FunktionenplotterFunkyPlot: Kostenloser und quelloffener Funktionenplotter für Linux und andere Betriebssysteme
StartseiteMatheForenUni-Lineare AlgebraElementarMatrizen
Foren für weitere Schulfächer findest Du auf www.vorhilfe.de z.B. Philosophie • Religion • Kunst • Musik • Sport • Pädagogik
Forum "Uni-Lineare Algebra" - ElementarMatrizen
ElementarMatrizen < Lineare Algebra < Hochschule < Mathe < Vorhilfe
Ansicht: [ geschachtelt ] | ^ Forum "Uni-Lineare Algebra"  | ^^ Alle Foren  | ^ Forenbaum  | Materialien

ElementarMatrizen: Frage (beantwortet)
Status: (Frage) beantwortet Status 
Datum: 18:30 Fr 25.06.2004
Autor: nevinpol

[hot] Notiz: [hot]

Also dies ist mein
[]vorletztes Übungsblatt, den ich am Montag morgen
abgeben muss. Ich muss in den letzten beiden Übungsblättern
alle Punkte sammeln, damit ich überhaupt die Klausur mitschreiben
kann. [verwirrt] Also deswegen bin ich jetzt hinter jedem Punkt hinterher...[anbet]


Hallo alle zusammen,

also ich mache dieses Wochenende : LA LA LA LA und am Ende bin ich
bestimmt ALALALLLALALLALALA :-)...

Aber nun zu meiner Frage:
Es geht um die Aufgabe 33 auf dem []Übungsblatt

Mein Problem ist; ich weiss nicht wie die Matrix [mm] $(E^j_i)_{kl}$ [/mm] aussieht.
Ich weiss dann auch nicht was dieses [mm] $E_n$ [/mm] bedeutet und was dieser [mm] $\lambda$ [/mm] in der [mm] $Q^j_i(\lambda)$ [/mm] tut.

Deswegen kann ich mir auch schonmal keinen Ansatz für die 33a) basteln..


Bitte [anbet] um Hilfe!

Vielen Dank
nevinpol



        
Bezug
ElementarMatrizen: Antwort
Status: (Antwort) fertig Status 
Datum: 18:56 Fr 25.06.2004
Autor: Stefan

Liebe Nevin!

Ich beantworte erst einmal nur deine Fragen, dann versuchst du die Aufgabe zunächst selber zu lösen, okay?

> also ich mache dieses Wochenende : LA LA LA LA und am Ende
> bin ich
>  bestimmt ALALALLLALALLALALA :-)...

;-)
  

> Aber nun zu meiner Frage:
>  Es geht um die Aufgabe 33 auf dem
> []Übungsblatt

  

> Mein Problem ist; ich weiss nicht wie die Matrix
> [mm](E^j_i)_{kl}[/mm] aussieht.

Das ist ein Eintrag. Die Matrix heißt [mm] $E_i^j$. [/mm] Sie besteht fast ausschließlich aus $0$en. Nur ein Eintrag ist gleich 1, nämlich der in der $i$-ten Zeile und $j$-ten Spalte.

>  Ich weiss dann auch nicht was dieses [mm]E_n[/mm] bedeutet und was
> dieser [mm]\lambda[/mm] in der [mm]Q^j_i(\lambda)[/mm] tut.

[mm] $E_n$ [/mm] ist die Einheitsmatrix, also die Diagonalmatrix, die auf der Diagonalen lauter $1$en hat. Der Rest sind $0$en.

Nun ist für $i [mm] \ne [/mm] j$: [mm] $Q_i^j(\lambda) [/mm] = [mm] E_n [/mm] + [mm] \lambda E_i^j$. [/mm]

Das bedeutet: [mm] $Q_i^j$ [/mm] hat auf der Diagonalen lauter $1$en, an der Stelle $(i,j)$ ein [mm] $\lambda$, [/mm] und besteht ansonsten nur aus $0$en.

Versuche es jetzt mal... Wir helfen dir dann schon, wenn du Probleme bekommst! :-)

Liebe Grüße
Stefan


Bezug
                
Bezug
ElementarMatrizen: Frage (beantwortet)
Status: (Frage) beantwortet Status 
Datum: 00:25 Sa 26.06.2004
Autor: nevinpol

Hallo Stefan,

also ich gehe es mal langsam ran obwohl ich eigentlich
schon knapp mit der Zeit bin...


>Das ist ein Eintrag. Die Matrix heißt [mm] $E_i^j$. [/mm] Sie besteht fast
>ausschließlich aus $0$en. Nur ein Eintrag ist gleich 1, nämlich
>der in der >$i$-ten Zeile und $j$-ten Spalte.

Also dann sieht [mm] $E_i^j$ [/mm] so aus.. ?

[mm] \begin{pmatrix} e_{11} & e_{12} & e_{13} & \cdots & e_{1j} \\ e_{21} & e_{22} & e_{23} & \cdots & e_{2j} \\ e_{31} & e_{32} & e_{33} & \cdots & e_{3j} \\ \vdots & \vdots & \vdots & \ldots & \vdots \\ e_{i1} & e_{i2} & e_{i3} & \cdots & e_{ij} \\ \end{pmatrix} [/mm]

Und [mm] $(E_i^j)_{kl}$ [/mm] sieht so aus...?

[mm] \begin{pmatrix} \delta_{1k} \cdot \delta_{1l} & \delta_{1k} \cdot \delta_{2l} & \delta_{1k} \cdot \delta_{3l} & \cdots & \delta_{1k} \cdot \delta_{jl} \\ \delta_{2k} \cdot \delta_{1l} & \delta_{2k} \cdot \delta_{2l} & \delta_{2k} \cdot \delta_{3l} & \cdots & \delta_{2k} \cdot \delta_{jl} \\ \delta_{3k} \cdot \delta_{1l} & \delta_{3k} \cdot \delta_{2l} & \delta_{3k} \cdot \delta_{3l} & \cdots & \delta_{3k} \cdot \delta_{jl} \\ \vdots & \vdots & \vdots & \ldots & \vdots \\ \delta_{ik} \cdot \delta_{1l} & \delta_{ik} \cdot \delta_{2l} & \delta_{ik} \cdot \delta_{3l} & \cdots & \delta_{ik} \cdot \delta_{jl} \\ \end{pmatrix} [/mm]

Und dann habe ich es so verstanden, dass der Eintrag
$ [mm] \delta_{ik} \cdot \delta_{jl}=1$ [/mm] ist und der Rest sind Nullen...?
Also wie folgt... Also ich denke mir schon, dass du die folgende
meinst, aber sind meine Überlegungen,
die dann zu diesem Matrix führen richtig oder meintest du was anders?

[mm] \begin{pmatrix} 0 & 0 & 0 & \cdots & 0 \\ 0 & 0 & 0 & \cdots & 0 \\ 0 & 0 & 0 & \cdots & 0 \\ \vdots & \vdots & \vdots & \ldots & \vdots \\ 0 & 0 & 0 & \cdots & 1 \\ \end{pmatrix} [/mm]

[mm] >$E_n$ [/mm] ist die Einheitsmatrix, also die Diagonalmatrix,
>die auf der Diagonalen lauter $1$en hat. Der Rest sind $0$en.

Dann ist [mm] $E_n$ [/mm] die Einheitsmatrix. Also:

[mm] \begin{pmatrix} 1 & 0 & 0 & \cdots & 0 \\ 0 & 1 & 0 & \cdots & 0 \\ 0 & 0 & 1 & \cdots & 0 \\ \vdots & \vdots & \vdots & \ldots & \vdots \\ 0 & 0 & 0 & \cdots & 1 \\ \end{pmatrix} [/mm]

>Nun ist für $i [mm] \ne [/mm] j$: [mm] $Q_i^j(\lambda) [/mm] = [mm] E_n [/mm] + [mm] \lambda E_i^j$. [/mm]

>Das bedeutet: [mm] $Q_i^j$ [/mm] hat auf der Diagonalen lauter $1$en,
>an der Stelle $(i,j)$ ein [mm] $\lambda$, [/mm] und besteht ansonsten nur aus $0$en.

Allllsoooo:

[mm] $E_n [/mm] + [mm] \lambda E_i^j$ [/mm]

[mm] \rightarrow \begin{pmatrix} 1 & 0 & 0 & \cdots & 0 \\ 0 & 1 & 0 & \cdots & 0 \\ 0 & 0 & 1 & \cdots & 0 \\ \vdots & \vdots & \vdots & \ldots & \vdots \\ 0 & 0 & 0 & \cdots & 1 \\ \end{pmatrix} + \lambda \cdot \begin{pmatrix} 0 & 0 & 0 & \cdots & 0 \\ 0 & 0 & 0 & \cdots & 0 \\ 0 & 0 & 0 & \cdots & 0 \\ \vdots & \vdots & \vdots & \ldots & \vdots \\ 0 & 0 & 0 & \cdots & 1 \\ \end{pmatrix} [/mm]

[mm] \rightarrow \begin{pmatrix} 1 & 0 & 0 & \cdots & 0 \\ 0 & 1 & 0 & \cdots & 0 \\ 0 & 0 & 1 & \cdots & 0 \\ \vdots & \vdots & \vdots & \ldots & \vdots \\ 0 & 0 & 0 & \cdots & 1 \\ \end{pmatrix} + \begin{pmatrix} 0 & 0 & 0 & \cdots & 0 \\ 0 & 0 & 0 & \cdots & 0 \\ 0 & 0 & 0 & \cdots & 0 \\ \vdots & \vdots & \vdots & \ldots & \vdots \\ 0 & 0 & 0 & \cdots & \lambda \\ \end{pmatrix} [/mm]


[mm] \rightarrow \begin{pmatrix} 1 & 0 & 0 & \cdots & 0 \\ 0 & 1 & 0 & \cdots & 0 \\ 0 & 0 & 1 & \cdots & 0 \\ \vdots & \vdots & \vdots & \ldots & \vdots \\ 0 & 0 & 0 & \cdots & 1+\lambda \\ \end{pmatrix} [/mm]

Nachdem ich mir das hier aufgeschrieben habe, habe ich mir jetzt überlegt...
warum du [mm] $\lambda$ [/mm] hast und ich [mm] $\lambda+1$... [/mm]
Also ich habe wahrscheinlich nicht mitgedacht, dass es für$i [mm] \ne [/mm] j$ gelten.
Bitte [anbet] der Fehler  ist doch das oder... bitte nichts mehr Kompliziertes :-)

Okay das liegt dann daran, dass die eine Matrix "länger" ist als die andere und
somit bei der Addition man überhaupt nicht zu dem Fall kommt, dass man
[mm] $1+\lambda$ [/mm] rechnet.

Also nach diesen Gedanken habe ich dann auch diese folgende Matrix als Ergebnis:

[mm] \rightarrow \begin{pmatrix} 1 & 0 & 0 & \cdots & 0 \\ 0 & 1 & 0 & \cdots & 0 \\ 0 & 0 & 1 & \cdots & 0 \\ \vdots & \vdots & \vdots & \ldots & \vdots \\ 0 & 0 & 0 & \cdots & \lambda \\ \end{pmatrix} [/mm]

Sooo und es war ja für $i [mm] \ne [/mm] j$: [mm] $Q_i^j(\lambda) [/mm] = [mm] E_n [/mm] + [mm] \lambda E_i^j$. [/mm]


Nach diesem ist doch Aufgabe33) trivial oder? Ich könnte doch einfach
Matrix_lambda mal Matrix_mu schreiben und das ist gleich Matrix_lambdamu

oder sollte ich da dass mit der Definition mit der Einheitsmatrix und so weiter ins Spiel bringen?

Und zu der Aufgabe33b) Wie beweist man denn invertierbarkeit?
Könnte ich einfach die Einheitsvektor mal Inverser nehmen und sagen tadaaa
das Q-lambda???

Vielen Dank
nevinpol


Bezug
                        
Bezug
ElementarMatrizen: Antwort
Status: (Antwort) fertig Status 
Datum: 02:09 So 27.06.2004
Autor: Stefan

Liebe Nevin!

> [mm] \begin{pmatrix} 0 & 0 & 0 & \cdots & 0 \\ 0 & 0 & 0 & \cdots & 0 \\ 0 & 0 & 0 & \cdots & 0 \\ \vdots & \vdots & \vdots & \ldots & \vdots \\ 0 & 0 & 0 & \cdots & 1 \\ \end{pmatrix} [/mm]

Nein, das ist so nicht richtig.  Warum steht der Eintrag $1$ gerade rechts unten? Er steht irgendwo, genauer gesagt im Eintrag $(i,j)$, also in der $i$-ten Zeile und $j$-ten Spalte, für ein $i [mm] \ne [/mm] j$,

Also auf keinen Fall auf der Diagonalen!


> Dann ist [mm]E_n[/mm] die Einheitsmatrix. Also:
>  
> [mm] \begin{pmatrix} 1 & 0 & 0 & \cdots & 0 \\ 0 & 1 & 0 & \cdots & 0 \\ 0 & 0 & 1 & \cdots & 0 \\ \vdots & \vdots & \vdots & \ldots & \vdots \\ 0 & 0 & 0 & \cdots & 1 \\ \end{pmatrix} [/mm]

Okay, das ist ja klar.  

> Allllsoooo:
>  
> [mm]E_n + \lambda E_i^j[/mm]
>  
> [mm] \rightarrow \begin{pmatrix} 1 & 0 & 0 & \cdots & 0 \\ 0 & 1 & 0 & \cdots & 0 \\ 0 & 0 & 1 & \cdots & 0 \\ \vdots & \vdots & \vdots & \ldots & \vdots \\ 0 & 0 & 0 & \cdots & 1 \\ \end{pmatrix} + \lambda \cdot \begin{pmatrix} 0 & 0 & 0 & \cdots & 0 \\ 0 & 0 & 0 & \cdots & 0 \\ 0 & 0 & 0 & \cdots & 0 \\ \vdots & \vdots & \vdots & \ldots & \vdots \\ 0 & 0 & 0 & \cdots & 1 \\ \end{pmatrix} [/mm]
>  
>
> [mm] \rightarrow \begin{pmatrix} 1 & 0 & 0 & \cdots & 0 \\ 0 & 1 & 0 & \cdots & 0 \\ 0 & 0 & 1 & \cdots & 0 \\ \vdots & \vdots & \vdots & \ldots & \vdots \\ 0 & 0 & 0 & \cdots & 1 \\ \end{pmatrix} + \begin{pmatrix} 0 & 0 & 0 & \cdots & 0 \\ 0 & 0 & 0 & \cdots & 0 \\ 0 & 0 & 0 & \cdots & 0 \\ \vdots & \vdots & \vdots & \ldots & \vdots \\ 0 & 0 & 0 & \cdots & \lambda \\ \end{pmatrix} [/mm]

>
> [mm] \rightarrow \begin{pmatrix} 1 & 0 & 0 & \cdots & 0 \\ 0 & 1 & 0 & \cdots & 0 \\ 0 & 0 & 1 & \cdots & 0 \\ \vdots & \vdots & \vdots & \ldots & \vdots \\ 0 & 0 & 0 & \cdots & 1+\lambda \\ \end{pmatrix} [/mm]
>  

Nein, siehe oben. Mache es bitte so, wie ich es beschrieben habe. Auf der Diagonalen $1$en und dann irgendwo, aber nicht auf der Diagonalen noch eine $1$ (beim Eintrag $(i,j)$ für $i [mm] \ne [/mm] j$).  



> Nachdem ich mir das hier aufgeschrieben habe, habe ich mir
> jetzt überlegt...
>  warum du [mm]\lambda[/mm] hast und ich [mm]\lambda+1[/mm]...
>  Also ich habe wahrscheinlich nicht mitgedacht, dass es
> für[mm]i \ne j[/mm] gelten.

Aha.


>  Bitte [anbet] der Fehler  ist doch das oder... bitte
> nichts mehr Kompliziertes :-)

Das war es.
  

> Okay das liegt dann daran, dass die eine Matrix "länger"
> ist als die andere und
>  somit bei der Addition man überhaupt nicht zu dem Fall
> kommt, dass man
>  [mm]1+\lambda[/mm] rechnet.

Häh???
  

> Also nach diesen Gedanken habe ich dann auch diese folgende
> Matrix als Ergebnis:
>  
> [mm] \rightarrow \begin{pmatrix} 1 & 0 & 0 & \cdots & 0 \\ 0 & 1 & 0 & \cdots & 0 \\ 0 & 0 & 1 & \cdots & 0 \\ \vdots & \vdots & \vdots & \ldots & \vdots \\ 0 & 0 & 0 & \cdots & \lambda \\ \end{pmatrix} [/mm]

Okay, du hast es doch nicht verstanden. Das [mm] $\lambda$ [/mm] darf nicht auf der Diagonalen stehen und hier wird auch nichts verlängert.  Das überlassen wir mal den Schönheitschirurgen. ;-)

> Sooo und es war ja für [mm]i \ne j[/mm]: [mm]Q_i^j(\lambda) = E_n + \lambda E_i^j[/mm].
>
>
>
> Nach diesem ist doch Aufgabe33) trivial oder?

Ja. Es gilt:

[mm] $Q_i^j(\lambda) \cdot Q_i^j(\mu)$ [/mm]

$ = [mm] (E_n [/mm] + [mm] \lambda E_i^j) \cdot (E_n [/mm] + [mm] \mu E_i^j)$ [/mm]

$ = [mm] E_n \cdot E_n [/mm] + [mm] (\lambda E_i^j) \cdot E_n [/mm] + [mm] E_n \cdot (\mu E_i^j) [/mm] + [mm] (\lambda E_i^j) \cdot (\mu E_i^j)$ [/mm]

$= [mm] E_n [/mm] + [mm] \lambda E_i^j [/mm] + [mm] \mu E_i^j$ [/mm]

$= [mm] E_n [/mm] + [mm] (\lambda [/mm] + [mm] \mu)E_i^j$ [/mm]

$= [mm] Q_i^j(\lambda [/mm] + [mm] \mu)$. [/mm]

Hierbei habe ich ausgenutzt, dass

[mm] $E_i^j \cdot E_i^j=0$ [/mm]

ist. Kannst du mir sagen, warum das so ist?

Nun ja, nur die $i$-te Zeile des ersten Faktors liefert etwas zum Produkt, und die die $j$-te Spalte des zweiten Faktors. Der einzige nichttriviale Eintrag der $i$-ten Zeile des ersten Faktors ist aber der $j$-te Eintrag, während der einzige nichttriviale Eintrag der $j$-ten Spalte des zweiten Faktors der $i$-te Eintrag ist. Wegen [mm] $i\ne [/mm] j$ hat also das Produkt keine von $0$ verschiedenen Einträge.

> Und zu der Aufgabe33b) Wie beweist man denn
> invertierbarkeit?

Man setzt einfach [mm] $\mu:= [/mm] - [mm] \lambda$ [/mm] und nutzt [mm] $Q_i^j(0)=E_n$ [/mm] aus.

Dann steht da:

[mm] $Q_i^j(\lambda) \cdot Q_i^j(-\lambda) [/mm] = [mm] Q_i^j(0)= E_n$, [/mm]

woraus die Behauptung folgt.

Liebe Grüße
Stefan


Bezug
Ansicht: [ geschachtelt ] | ^ Forum "Uni-Lineare Algebra"  | ^^ Alle Foren  | ^ Forenbaum  | Materialien


^ Seitenanfang ^
www.schulmatheforum.de
[ Startseite | Forum | Wissen | Kurse | Mitglieder | Team | Impressum ]